Quanten.de Diskussionsforum  

Zur?ck   Quanten.de Diskussionsforum > Quantenmechanik, Relativitätstheorie und der ganze Rest.

Hinweise

Quantenmechanik, Relativitätstheorie und der ganze Rest. Wenn Sie Themen diskutieren wollen, die mehr als Schulkenntnisse voraussetzen, sind Sie hier richtig. Keine Angst, ein Physikstudium ist nicht Voraussetzung, aber man sollte sich schon eingehender mit Physik beschäftigt haben.

Antwort
 
Themen-Optionen Ansicht
  #1  
Alt 31.08.09, 23:33
ZeroPointEnergy ZeroPointEnergy ist offline
Newbie
 
Registriert seit: 31.08.2009
Beitr?ge: 1
Standard Doppelspaltexperiment mit verschränkten Photonen

Hallo Zusammen

Ich habe eine Frage die mich seit einiger Zeit wurmt über ein Experiment über welches ich in Spektrum der Wissenschaft gelesen habe.

Zu meiner Person: Ich habe keinen Abschluss in Physik oder ähnlichem, ich komme aus der Informatik, beschäftige mich aber seit der Schulzeit intensiv mit Populärwissenschaftlicher Literatur quasi als Hobby.

Nun hatte ich teilweise wirklich das Gefühl ich würde wenigstens die Umrisse des ganzen ein wenig erkenne, besagtes Experiment lässt mich jedoch je länger je mehr daran zweifeln ich hoffe das mir jemand aus diesem Forum auf die Sprünge helfen kann.

Zum Experiment:
Bei besagtem Experiment handelt es sich um ein zweifaches Doppelspalt Experiment. Es gibt zwei Strecken mit jeweils einem Doppelspalt und dem Detektor dahinter. Nun wird ein verschränktes Photonenpaar erzeugt und dieses wird auf jeweils einer Strecke mit Doppelspalt abgeschossen.

Nun verhällt es sich ja so das am Detektor solange ein Interferenzmuster entsteht wie die Wellenfunktion der Photonen intakt ist. Sobald ich eine Messung vornehmne kollabiert die Wellenfunktion und das Ergebniss am Detektor hat eine Teilchen charakteristik.

Was passiert aber mit dem verschränkten Teilchen? In diesem Experiment wurde beschrieben das sich die Messung auch auf das verschränkte Photon auswirkt, da dessen Wellenfunktion gleichzeitig kollabiert oder eben auch nicht falls wir die Messung weglassen. Die Messung auf der einen Strecke beeinflusst also das Ergebniss am Detektor auf der anderen Seite.

Ist das soweit richtig?

Nun ist mir durchaus bewusst das sich durch die Verschränkung keine Informationen übertragen lassen, da die Zustände welche die Photonen annehmen jeweils zufällig sind. In der Informatik würden wir daraus wohl einen Räumlich getrennten synchronen Zufallszahlengenerator bauen . Ist wohl das was in der Quantenkryptopgraphie benutzt wird.

Aber was ist mit der Wellenfunktion an sich? Wenn ich durch die Messung auf der einen Strecke beeinflussen kann das die Wellenfunktion auf der anderen Strecke kollabiert oder eben auch nicht, kann ich dann nicht auf diesem Weg Informationen übertragen??

0: Sender (keine Messung) -> Empfänger (Interferenzmuster)
1: Sender (Messung) -> Empfänger (kein Interferenzmuster)

Ja, ihr seit sicher schon alle am schreien weil ich absolut nichts verstanden habe :-) Leider sehe ich meinen Überlegungsfehler nicht oder villeicht ist ja das Experiment an sich Mist oder ich habe nicht alle Informationen. Könnt ihr mir helfen?

Sonst fang ich noch an das Experiment nachzubauen und ich weis schon was dabei rauskommt: fail. Und nachher bin ich sicher auch nicht schlauer
Mit Zitat antworten
  #2  
Alt 01.09.09, 13:39
Matz Matz ist offline
Newbie
 
Registriert seit: 27.08.2009
Beitr?ge: 19
Idee AW: Doppelspaltexperiment mit verschränkten Photonen

Hallo!

Hoffentlich erzähl ich jetzt keinen allzu großen Mist
Ich glaube es ist so:
Der Verschränkung bezieht sich auf den Spin-Anteil der Wellenfunktion, d.h. eine Messung der Polarisation des einen Photons legt die des anderen fest. Für die Interferenz ist aber der Orts-Anteil entscheidend. Wenn du mit Messung meinst, dass der Orts-Teil kollabiert, führt das dazu, dass das Interferenzmuster verschwindet. Die Polarisation wird dadurch aber nicht festgelegt, und auch nicht die Ortswellenfunktion des anderen Photons, weshalb sich trotzdem am anderen Doppelspalt ein Interferenzmuster ergibt.
Grüße,

Matz
Mit Zitat antworten
  #3  
Alt 02.09.09, 21:10
Benutzerbild von Gandalf
Gandalf Gandalf ist offline
Singularität
 
Registriert seit: 01.05.2007
Beitr?ge: 1.080
Standard AW: Doppelspaltexperiment mit verschränkten Photonen

Zitat:
Zitat von ZeroPointEnergy Beitrag anzeigen

Was passiert aber mit dem verschränkten Teilchen? In diesem Experiment wurde beschrieben das sich die Messung auch auf das verschränkte Photon auswirkt, da dessen Wellenfunktion gleichzeitig kollabiert oder eben auch nicht falls wir die Messung weglassen. Die Messung auf der einen Strecke beeinflusst also das Ergebniss am Detektor auf der anderen Seite.

Ist das soweit richtig?
Hi ZPE!

Das ist mehrfach falsch. Ich denke hier wurde verschiedenes durcheinandergebracht. Unabhängig das ich nicht der richtige bin, der Dir erklären können 'wollte', dass 'eine Funktion zusammenbricht' (So etwas wurde noch NIE nachgewiesen und ich bezeichne diese "Betrachtung" ausdrücklich und begründet als 'Schwachfug'). Und das mit der "Beeinflussung" ist auch nicht etwas was man etwa mit der Beeinflussung einer Billardkugel durch einen Queue vergleichen könnte.

Sorry, dass ich heute keine Lust dazu habe (und andere anscheinend auch nicht), hier näher darauf einzugehen. Da das DS hier regelmäßig thematisiert wird, empfehle ich Dir erst mal Dich ein bischen hier einzulesen, vielleicht klären sich manche Fragen (und falsche Vorstellungen) dann allein.

Viele Grüße
__________________

Warum soll sich die Natur um intellektuelle Wünsche kümmern, die "Objektivität" der Welt des Physikers zu retten? Wolfgang Pauli
Mit Zitat antworten
  #4  
Alt 02.09.09, 22:45
Benutzerbild von Optimist71
Optimist71 Optimist71 ist offline
Aufsteiger
 
Registriert seit: 01.05.2007
Ort: Oslo (Norwegen)
Beitr?ge: 67
Standard AW: Doppelspaltexperiment mit verschränkten Photonen

Hallo ZPE,

Zitat:
Zitat von ZeroPointEnergy Beitrag anzeigen
Zum Experiment:
Bei besagtem Experiment handelt es sich um ein zweifaches Doppelspalt Experiment. Es gibt zwei Strecken mit jeweils einem Doppelspalt und dem Detektor dahinter. Nun wird ein verschränktes Photonenpaar erzeugt und dieses wird auf jeweils einer Strecke mit Doppelspalt abgeschossen.

Nun verhällt es sich ja so das am Detektor solange ein Interferenzmuster entsteht wie die Wellenfunktion der Photonen intakt ist. Sobald ich eine Messung vornehmne kollabiert die Wellenfunktion und das Ergebniss am Detektor hat eine Teilchen charakteristik.
Was ist denn der Doppelspaltversuch? Richtig, eine Messung! Fuer jedes einzelne Photon wird eine Positionsmessung durchgefuehrt (mit dem Messergebnis in Form eines Punktes auf der Photoplatte). Ein Interferenzmuster wird erst sichtbar, wenn Du den Doppelspalt mit sehr vielen Photonen bombadierst (Bestrahlung mit monochromatischem Licht). Da Du allerdings in Deinem Versuch nur jeweils ein Photon auf jeden Spalt schiesst, wirst Du auch jeweils nur einen Punkt auf der Photoplatte sehen und kein Interferenzmuster.

Zitat:
Was passiert aber mit dem verschränkten Teilchen? In diesem Experiment wurde beschrieben das sich die Messung auch auf das verschränkte Photon auswirkt, da dessen Wellenfunktion gleichzeitig kollabiert oder eben auch nicht falls wir die Messung weglassen. Die Messung auf der einen Strecke beeinflusst also das Ergebniss am Detektor auf der anderen Seite.

Ist das soweit richtig?
Nein. Dass die beiden Photonen verschraenkt sind bedeutet, dass sie mit einer gemeinsamen Zustandsfunktion beschrieben werden.

Das mit dem "Kollaps" der Wellenfunktion ist eigentlich ein veralteter Begriff, der mehr verwirrt als dass er nutzt. Eine Messung ist eine Wechselwirkung. Diese Wechselwirkung hat eine Dauer > 0. Was "waehrend" dieser Messung passiert, ist unbekannt. Was da auch immer passiert, unmittelbar nach der Messung hat die Wellenfunktion eine Form, die man einen Eigenzustand nennt. In solch einem Eigenzustand wird einer der moeglichen Messwerte fuer die gemessene Observable sicher (mit Wahrscheinlichkeit 1) gemessen, fuer die anderen moeglichen Messwerte ist die Messwahrscheinlichkeit in diesem Eigenzustand Null.

Die "Beeinflussung" zweier Teilchen im gleichen Quantensystem ist dabei nur reine Statistik: Das Betragsquadrat der Wellenfunktion ist eine Wahrscheinlichkeitsdichtefunktion. Ist sie einmal normiert (Integral der Wahrscheinlichkeitsdichtefunktion ueber den gesamten Raum = 1), so ist sie es immer. Ein Zweiteilchenproblem mit gemeinsamer Zustandsfunktion quantenmechanisch zu beschreiben ist zumindest fuer massenbehaftete Teilchen kein Problem (fuer Photonen kann ich's nicht, was nicht heissen soll, dass es nicht geht). Eine Messung im gemeinsamen Quantensystem an einem beliebigen Ort, z.B. Detektion eines der beiden Teilchen, hat eine Aenderung der gemeinsamen Zustandsfunktion und damit auch der Wahrscheinlichkeiten fuer die moeglichen Messergebnisse des zweiten Teilchens zur Folge.

Fuer Informationsuebertragung ist ein System mit zwei verschraenkten Teilchen nicht geeignet.

-- Optimist
Mit Zitat antworten
  #5  
Alt 07.09.09, 16:24
Matz Matz ist offline
Newbie
 
Registriert seit: 27.08.2009
Beitr?ge: 19
Frage AW: Doppelspaltexperiment mit verschränkten Photonen

Zitat:
Zitat von Gandalf Beitrag anzeigen
Hi ZPE!
Unabhängig das ich nicht der richtige bin, der Dir erklären können 'wollte', dass 'eine Funktion zusammenbricht' (So etwas wurde noch NIE nachgewiesen und ich bezeichne diese "Betrachtung" ausdrücklich und begründet als 'Schwachfug').
Hey!

Sorry, dass ich das nochmal aufrolle, aber was ist denn so schlecht an der Formulierung, dass "eine Funktion zusammenbricht"? Man spricht doch vom "Kollaps der Wellenfunktion"; und es ist doch tatsächlich so, dass eine Funktion |Ψ-> = Σc(n)|φ(n)>, die vor der Messung aus einer Linearkombination der Eigenzustände |φ(n)> besteht, bei der Messung "zusammenbricht" und nach der Messung nur noch aus einem Eigenzustand |Ψ+> = |φ(m)> besteht?

Matz
Mit Zitat antworten
  #6  
Alt 07.09.09, 17:19
Benutzerbild von JoAx
JoAx JoAx ist offline
Singularität
 
Registriert seit: 05.03.2009
Beitr?ge: 4.324
Standard AW: Doppelspaltexperiment mit verschränkten Photonen

Hallo Matz,

bevor der "Streit" zwischen KD und VWT auch in diesem Thread ausbricht , schlage ich vor, du liest diesen hier:

http://www.quanten.de/forum/showthread.php5?t=1102

durch, bzw. stellst deine Frage dort. Dann bleibt das Thema kompakt


Gruss, Johann
Mit Zitat antworten
  #7  
Alt 07.09.09, 18:23
Eyk van Bommel Eyk van Bommel ist offline
Singularität
 
Registriert seit: 08.07.2007
Beitr?ge: 3.798
Standard AW: Doppelspaltexperiment mit verschränkten Photonen

Hi Optimist,
Zitat:
Diese Wechselwirkung hat eine Dauer > 0.
Woher hast du das? Hast du eine Begründung für diese Aussage?

Gruß
EVB
__________________
Phantasie ist wichtiger als Wissen, denn Wissen ist begrenzt. A.E
Mit Zitat antworten
  #8  
Alt 07.09.09, 18:38
Benutzerbild von Gandalf
Gandalf Gandalf ist offline
Singularität
 
Registriert seit: 01.05.2007
Beitr?ge: 1.080
Standard AW: Doppelspaltexperiment mit verschränkten Photonen

Zitat:
Zitat von Eyk van Bommel Beitrag anzeigen
Hi Optimist,

Zitat:
Diese Wechselwirkung hat eine Dauer > 0.
Woher hast du das? Hast du eine Begründung für diese Aussage?

Gruß
EVB

... wollte ich auch schon mal fragen...
__________________

Warum soll sich die Natur um intellektuelle Wünsche kümmern, die "Objektivität" der Welt des Physikers zu retten? Wolfgang Pauli
Mit Zitat antworten
  #9  
Alt 08.09.09, 09:19
Benutzerbild von Optimist71
Optimist71 Optimist71 ist offline
Aufsteiger
 
Registriert seit: 01.05.2007
Ort: Oslo (Norwegen)
Beitr?ge: 67
Standard AW: Doppelspaltexperiment mit verschränkten Photonen

Zitat:
Zitat von Eyk van Bommel Beitrag anzeigen
Hi Optimist,

Zitat:
Diese Wechselwirkung hat eine Dauer > 0.
Woher hast du das? Hast du eine Begründung für diese Aussage?

Gruß
EVB
Hallo EVB, hallo Gandalf,

Diese Aussage kann ich in der Tat nicht wirklich begruenden. Streicht diesen Satz also aus meinem Posting. Danke fuer's Nachfragen!

Wichtig war mir nur darzustellen, was ein Eigenzustand ist, und wann ein Quantensystem sich in einem Eigenzustand zu einem Operator befindet - naemlich unmittelbar nach der Messung bzw. Wechselwirkung. Vor der Messung entwickelt sich die Wellenfunktion gemaess der Schroedingergleichung. Was "waehrend der Messung" geschieht und wie lange diese dauert, darueber macht die Quantenmechanik meines Erachtens nach keine Aussage.

-- Optimist
Mit Zitat antworten
  #10  
Alt 08.09.09, 14:15
Benutzerbild von Gandalf
Gandalf Gandalf ist offline
Singularität
 
Registriert seit: 01.05.2007
Beitr?ge: 1.080
Standard AW: Doppelspaltexperiment mit verschränkten Photonen

Zitat:
Zitat von Optimist71 Beitrag anzeigen
Hallo EVB, hallo Gandalf,

Diese Aussage kann ich in der Tat nicht wirklich begruenden. Streicht diesen Satz also aus meinem Posting. Danke fuer's Nachfragen!

Wichtig war mir nur darzustellen, was ein Eigenzustand ist, und wann ein Quantensystem sich in einem Eigenzustand zu einem Operator befindet - naemlich unmittelbar nach der Messung bzw. Wechselwirkung. Vor der Messung entwickelt sich die Wellenfunktion gemaess der Schroedingergleichung. Was "waehrend der Messung" geschieht und wie lange diese dauert, darueber macht die Quantenmechanik meines Erachtens nach keine Aussage.

-- Optimist

... ich hatte mich meinerseits eher an dem Begriff "Wechselwirkung" gestört (bzw. störe mich immer noch), - da es sich ja wohl um eine Verschränkung handeln muss... (und zur Verschränkung gibt es Aussagen innerhalb der QT)


Grüße
__________________

Warum soll sich die Natur um intellektuelle Wünsche kümmern, die "Objektivität" der Welt des Physikers zu retten? Wolfgang Pauli
Mit Zitat antworten
Antwort

Lesezeichen

Themen-Optionen
Ansicht

Forumregeln
Es ist Ihnen nicht erlaubt, neue Themen zu verfassen.
Es ist Ihnen nicht erlaubt, auf Beitr?ge zu antworten.
Es ist Ihnen nicht erlaubt, Anh?nge hochzuladen.
Es ist Ihnen nicht erlaubt, Ihre Beitr?ge zu bearbeiten.

BB-Code ist an.
Smileys sind an.
[IMG] Code ist an.
HTML-Code ist aus.

Gehe zu


Alle Zeitangaben in WEZ +1. Es ist jetzt 12:02 Uhr.


Powered by vBulletin® Version 3.8.8 (Deutsch)
Copyright ©2000 - 2024, vBulletin Solutions, Inc.
ScienceUp - Dr. Günter Sturm